2.4x-15=1
3. 2y+3= -11
4.2y+7= -7
5. 3w+3=3

Answers

Answer 1

Answer:

2. x=4

3. y= -7

4. y = -7

5. w = 0

Step-by-step explanation:

2.

4x-15=1

4x=16

x=4

3.

2y+3= -11

2y=-14

y= -7

4.

2y+7= -7

2y = -14

y = -7

5.

3w+3=3

3w = 0

w = 0

Answer 2
2. x=4

step-by-step:
4x-15=1
4x=16 (add 15 to both sides)
x=4 (divide both sides by 4)

3. y=-7

step-by-step:
2y+3=-11
2y=-14 (subtract 3 from both sides)
y=-7 (divide both sides by 2)

4. y=-7

step-by-step:
2y+7=-7
2y=-14 (subtract 7 from both sides)
y=-7 (divide both sides by 2)

5. w=0

step-by-step:
3w+3=3
3w=0 (subtract 3 from both sides)
w=0 (divide both sides by 3)

hope this helps!

Related Questions

One leg of a right triangle is 7 inches longer than the other leg, and the hypotenuse is 35 inches. Find the lengths of the legs of the triangle.

Answers

Answer: 21, 28

Step-by-step explanation:

Side #1 = xSide #2 = x + 7Hypotenuse = 35

Use the Pythagorean Theorem [tex]a^{2}+b^{2}=c^{2}[/tex]:

a = xb = x + 7c = 35

Substitute in the values & solve:

[tex]x^{2}+(x+7)^{2}=35^{2}\\x^{2}+x^{2}+14x+49=1225\\2x^{2}+14x+49-1225=0\\2x^{2}+14x-1176=0\\2(x^{2}+7x-588)=0\\2(x + 28)(x - 21)=0\\x_{1}=-28, x_{2}=21[/tex]

-28 is not a possible solution since you can't have negative inches...

a = x = 21b = x + 7 = 21 + 7 = 28c = 35

sue has 18 pieces of candy
tony has 18 pieces of candy
sue then gives some to tony
sue then eats five of hers
tony eats half of his
write the expressions for the number of pieces candy sue and tony now have?

Answers

Answer:

Sue candy = 13 - x

Tony candy = 9 + 1/2x

Step-by-step explanation:

Sue candy = 18

Tony candy = 18

Let x = some candy gives to tony

Sue candy = 18 - x

Tony candy = 18 + x

sue then eats five of hers

Sue candy = 18 - x - 5

= 13 - x

tony eats half of his

Tony candy = 1/2(18 + x)

= 18/2 + x/2

= 9 + 1/2x

Expressions for the number of pieces candy sue and tony now have:

Sue candy = 13 - x

Tony candy = 9 + 1/2x

What is the slope of the line? What is the y-intercept of the line? y = -3x + 4

Answers

Answer:

slope= -3/1

y-intercept= 4

Answer:

m = -3

y intercept = 4

Step-by-step explanation:

The given equation of the line is ,

[tex]\implies y = -3x+4[/tex]

We know that the Standard equation of Slope Intercept Form of the line is,

[tex]\implies y = mx + c[/tex]

Where ,

m is slope c is y intercept

On comparing to the Standard form of the line we get ,

[tex]\implies Slope = -3 [/tex]

[tex]\implies y - intercept= 4 [/tex]

Drag the operator to the correct location on the image.
Which operation results in a binomial?

Answers

The correct answer is to drag The Plus sign (+)

What is an Operator?

This has to do with the use of symbols to denote mathematical equations such as addition, subtraction, etc.

Hence, we can see that the correct position to put the operator on the image to result in a binomial is to drag the plus sign (+) so that the equations can be solved,.

Read more about operators here:

https://brainly.com/question/25974538

#SPJ2

Answer:.

Step-by-step explanation:

log8-log4 ÷ log4-log2=





Answers

The answer is log(4)-1

are cans of bottles packaged at a faster rate

Answers

Answer:

yes.. they are

Step-by-step explanation:

because

A number ending in ___ is never a perfect square. ​

Answers

Answer:

2, 3, 7 or 8

Step-by-step explanation:

one more question
(-8)+___=-17

Answers

Answer:

-9

Step-by-step explanation:

Answer:

-9

Step-by-step explanation:

-8 +___= - 17

___=-17 +8

___=-9

=-9

Recipe ingredients remain jn a constant ratio no matter how many serving are prepared. Which table shows a possible ratio table for ingredients C and Y for the given number of servings

Answers

Answer:

The last table (the bottom one)

Step-by-step explanation:

The ingredients having the same ratio means that, for every number of servings, we should have:

Y/X = constant.

So, for the first table when we have 1 serving, the quotient is:

Y/X = 2/1 = 2

when we have two servings:

Y/X = 3/2 = 1.5

The ratios are different.

Then this is not the correct option.

For the second table, when we have 1 serving the ratio is:

Y/X = 2/1 = 2

when we have two servings:

Y/X = 4/2 = 2

when we have 3 servings:

Y/X = 8/3 = 2.66

This is not the correct option.

For the third table:

1 serving:

Y/X = 2/1 = 2

2 sevings

Y/X = 3/2 = 1.5

This is not the correct option.

fourth table:

1 serving:

Y/X = 2/1 = 2

2 servings

Y/X = 4/2 = 2

3 servings

Y/X = 8/4 = 2

Here we can see that the ratio is always the same, then the ratio remains constant.

This is the table that shows a possible ratio for ingredients X and Y,

Solve for x. Round to the nearest tenth, if necessary.

Answers

Answer:

11.3

Step-by-step explanation:

first we find angle F.

remember, all angles in a triangle always sum up to 180 degrees.

so,

F = 180 - 90 - 61 = 29 degrees

now we use the law of sines.

EF/sin(D) = ED/sin(F) = DF/sin(E)

DF = x

sin(E) = sin(90) = 1

5.5/sin(29) = x/1 = x

x = 11.3

Identify a horizontal or vertical stretch or compression of the function by observing the equation of the function .

Answers

Bdbxbcjncnxndnsnns Identify a horizontal or vertical stretch or compression of the function by observing the equation of the function .hdbdbebqb

Twenty of the 50 digital video recorders (DVRS) in an inventory are known to be detective What is the probability that a randomly selected item is defective?​

Answers

Answer:

3/5

Step-by-step explanation:

Remember that odds for plus odds against must be 1. Therefore, 20 out of 50 of the DVRs are defective, so 30 out of 50 would work.

30/50

simplifies into 3/5 so that would be the probability of you picking out a DVR that works, and the probability of picking one that does not work would be the leftover 2/5 . Or, if you're answering with a percent, you would have a 60% chance of picking a working DVR.

I hope this helps, I know the explanation isn't really clear, but I can't really think of another way to explain it.

What is the equation of the line??

I NEED ANSWERS NOW PLEASEEEEE

for 20 points pls people help me out

Answers

Answer:

x = 0

Step-by-step explanation:

This is a vertical line, in fact the y- axis

The equation of a vertical line is

x = c

where c is the value of the x- coordinates the line passes through

All of the x- coordinates on the y- axis are zero, then

x = 0 ← equation of line

3 questions only pls help gota finish tofday

Answers

5a) 3x3x13

b) 3x7x7

c)11x5x2x2

d)3x3x3x5x2

6a)2x2x2x131

7a)5x5x2x2

b) 3x2x17

c)5x2x2x2x2x2x2

d)5x5x5x3

e) 2x2x103

f)5x3x137

g) 2x2x2x2x2x2x2x2x2

h)5x5x7x7x3

estimate the solution to the following system of equations by graphing.
3x+5y=14
6x-4y=9

A. ( 4/3,5/2)
B. ( 7/3, -7/2 )
C. ( -5/2, -7/2)
D. (5/2, 4/3) ​

Answers

Answer:

(101/42, 19/14)

Step-by-step explanation:

Given the following equations

3x+5y=14 ..... 1 * 2

6x-4y=9 ... 2 * 1

__________________

6x+10y=28

6x-4y=9  

Subtract

10y+4y = 28 - 9

14y = 19

y = 19/14

Get the value of x

Since 3x+5y = 14

3x+5(19/14) = 14

3x + 95/14 = 14

3x = 14 - 95/14

3x = 101/14

42x = 101

x = 101/42

Hence the solution is (101/42, 19/14)

For the function G defined by G(x)=5x+3, find G(r+5)

Answers

Given function:

g(x) = 5x + 3

Find

g(r+5)

Substitute x with r = 5:

g(r + 5) = 5(r + 5) + 3 = 5r + 25 + 3 = 5r + 28

Answer:

G ( r + 5 ) = 5r + 28

Step-by-step explanation:

Given ;

G ( x ) = 5x + 3

To Find :-

G ( r + 5 )

Solution :-

plug r + 5 as x in the function.

G ( r + 5 ) = 5 ( r + 5 ) + 3

distribute 5

G ( r + 5 ) = 5r + 25 + 3

combine like terms

G ( r + 5 ) = 5r + 28

8^5 = 2^2m+3


Solve m

Answers

Answer:

[tex]m=6[/tex]

Step-by-step explanation:

Exponent properties:

We can use exponent property [tex]a^{b^c}=a^{(b\cdot c)}[/tex] to solve this problem.

Rewrite [tex]8[/tex] as [tex]2^3[/tex], then apply exponent property [tex]a^{b^c}=a^{(b\cdot c)}[/tex] to simplify:

[tex]2^{3^5}=2^{2m+3},\\2^{15}=2^{2m+3}[/tex]

If [tex]a^b=a^c[/tex], then [tex]b=c[/tex], because of log property [tex]\log a^b=b\log a[/tex]. Using this log property, you can take the log of both sides and divide by [tex]\log a[/tex] to get [tex]b=c[/tex]

Therefore, we have:

[tex]15=2m+3[/tex]

Subtract 3 from both sides:

[tex]12=2m[/tex]

Divide both sides by 6:

[tex]m=\frac{12}{2}=\boxed{6}[/tex]

Alternative:

Given [tex]8^5=2^{2m+3}[/tex], to move the exponent down, we'll use log properties.

Start by simplifying:

[tex]\log 32,768=2^{2m+3}[/tex]

Take the log of both sides, then use log property [tex]\log a^b=b\log a[/tex] to move the exponent down:

[tex]\log(32,768)=\log 2^{2m+3},\\\log (32,768)=(2m+3)\log 2[/tex]

Divide both sides by [tex]\log2[/tex]:

[tex]2m+3=\frac{\log (32,768)}{\log(2)}[/tex]

Subtract 3 from both sides:

[tex]2m=\frac{\log (32,768)}{\log(2)}-3[/tex]

Divide both sides by 2:

[tex]m=\frac{\log (32,768)}{2\log(2)}-\frac{3}{2}=\boxed{6}[/tex]

t=29pi/6
1. find the reference number
2. find the point on the unit circle
3. 6 trig functiond

Answers

the correct answer is C....I hope

Someone please help me ASAP

Answers

Step-by-step explanation:

a vector multiplied by a scalar is equal to it's image. The expression above gives an equation and after solving, it gives you the image

someone help me for this algebra task please

Answers

Answer:

200

Step-by-step explanation:

Substitute 15 for y

[tex] \frac{1}{5} x - \frac{2}{3} (15) = 30[/tex]

[tex] \frac{1}{5} x - 10 = 30[/tex]

[tex] \frac{1}{5} x = 40[/tex]

[tex]x = 200[/tex]

WILL GIVE BRAINLIEST AND 50 POINTS!
Determine the more basic function that has been shifted, reflected, stretched, or compressed.
m(x)= 2√x-5 -2

Answers

Answer: The function M if derived from the integral of the derivative of the function. This is valid by the fundamental theorem of calculus.

M(x)=43x32−7x+C

Step-by-step explanation: not 100 about this but it's what I got

On a coordinate plane, a line goes through (negative 3, negative 4) and (3, 0).
What are the necessary criteria for a line to be perpendicular to the given line and have the same y-intercept?

The slope is Negative three-halves and contains the point (0, 2).
The slope is Negative two-thirds and contains the point (0, −2).
The slope is Three-halves and contains the point (0, 2).
The slope is Negative three-halves and contains the point (0, −2

Answers

Answer:

The slope is Negative three-halves and contains the point (0, 2).

Step-by-step explanation:

(-3,-4)(3,0)

M= -4/-6 = 2/3

⊥M = -3/2

0 = 2/3(3) + B

B=2

Find the measure of the missing angle using the exterior angle sum theorm.

Answers

Answer:

85°

Step-by-step explanation:

The exterior angle of a triangle is=sum of the opposite interior angles

So

? °=45°+40°

Which point is in the solution set of this system inequalities?

A. (0,0)

B. None of these

C. (5,1)

D. (3,7)

Answers

Answer:

B

Step-by-step explanation:

To find which ordered pairs are solutions to the inequalities we can simply plug in the x and y values of the ordered pairs into the inequalities and if the equation is true for both inequalities then the ordered pair is a solution to the inequalities.

For (0,0)

x = 0

y = 0

y > x + 5

Substitute 0 for y and x

0 > 0 + 5

Simplify right side

0 > 5

The inequality is not true as 5 is greater than 0, not less than. So immediately we can eliminate answer choice A.

For (5,1).

x = 5

y = 1

y > x + 5

Substitute 5 for x and 1 for y

1 > 5 + 5

Simplify right side

1 > 10

Again, the equation is not true as 1 is not greater than 10. This means that c cannot be the answer

For (3,7)

x = 3

y = 7

y > x + 5

Substitute 3 for x and y for 7

7 > 3 + 5

Simplify right side

7 > 8

7 is not greater than 8 meaning that (3,7) cannot be a solution to the inequalities

None of the ordered pairs created true equations hence the answer is B

Please help me with this one I seriously suck at math

Answers

Answer:

194

Step-by-step explanation:

Rectangles :

7 x 10 = 70

6 x 10 = 60

4 x 10 = 40

Triangles

[tex]\frac{6x4}{2}[/tex] = 12   (two of them = 24)

70 + 60 + 40 + 24 = 194

P(x) is a polynomial. here are a few values of p(x).
P(-5) = - 2
P(-3) = 6
P(3) = 7
P(5) = -1
What is the remainder when P(x) is divided by (x+5)?
What is the remainder when P(x) is divided by (x-3)?

Answers

Given:

Values of a polynomial P(x).

[tex]P(-5)=-2[/tex]

[tex]P(-3)=6[/tex]

[tex]P(3)=7[/tex]

[tex]P(5)=-1[/tex]

To find:

The remainder when P(x) is divided by (x+5).

The remainder when P(x) is divided by (x-3).

Solution:

If a polynomial P(x) is divided by (x-a), then the remainder is P(a).

If the polynomial P(x) is divided by (x+5), then the remainder is P(-5).

[tex]P(-5)=-2[/tex]

Therefore, the remainder is -2 when P(x) is divided by (x+5).

If the polynomial P(x) is divided by (x-3), then the remainder is P(3).

[tex]P(3)=7[/tex]

Therefore, the remainder is 7 when P(x) is divided by (x-3).

Please help. I don’t understand I rlly appreciate it if you help!

Answers

Hi there!

We know that the angles ∠B and ∠A are supplementary because they are both interior angles. Therefore:

180 = ∠A + ∠B

Rewrite:

180 = (6x - 48) + (4x + 38)

Combine like terms:

180 = 10x - 10

Solve for x:

190 = 10x

x = 19

Solve for ∠B by plugging in this value of x:

∠B = 4(19) + 38

∠B = 76 + 38

∠B = 114

if f(x) = 2x/7 +4, which of the following is the inverse of f(x)?

Answers

Answer:

The answer is C

Step-by-step explanation:

Switch x and y

[tex]x = \frac{2y}{7} + 4[/tex]

Solve for y

[tex]y = \frac{7(x - 4)}{2} [/tex]

what is the length of segment DC? no links.

Answers

Answer:

33 units

Step-by-step explanation:

AB = BC

2x + 7 = 23

2x = 16

x = 8

DC = DA

DC = 4x + 1

DC = 4(8) + 1

DC = 33

In In 5x + In In (x - 1) = 2

Answers

Answer:

exact form: x=-1/2

decimal form: x=-0.5

Other Questions
A massless, hollow sphere of radius R is entirely filled with a fluid such that its density is p. This same hollow sphere is now compressed so that its radius is R/2, and then it is entirely filled with the same fluid as before. As such, what is the density of the compressed sphere? a. 8p b. p/8 c. p/4d. 4p Help ASAP only right answers only no spam dont answer if you dont know how is the sovereignty of a nation-state limited?A. it cannot enforce its laws outside its own territory B. it cannot pass laws that contradict the laws of its regionsC. the head of state is not required to follow the rule of lawD. its citizens may or may not consider its authority legitimate Being the Sales Manager of a company you have to hire more salesperson for the company to expand the sales territory. Kindly suggest an effective Recruitment and Selection Process to HR by explaining the all the stages in detail. Preocupada com o aumento da tarifa na conta de luz, uma pessoa resolve economizar diminuindo o tempo de banho de 20 para 15 minutos. Seu chuveiro possui as seguintes especificaes: 4200 W e 220V. Sabendo que o kWh custa R$0,30, a economia feita em 10 dias foi de aproximadamente Mark gathered data about the number of pink and red flowers that bloomed on several of his flowering shrubs. The scatter plot shows the data he gathered and the line of best fit. GET 95 POINTS Let f(x) = 1/x and g(x)=x + 6x. Whattwo numbers are not in the domain of fg?Separate your answers with a comma. Read the passage.Why did the officers ask Kai what the chairs in his home looked like?A They had seen the Chinese poems engraved on the walls.B They thought it was a good question for a 12-year-old.C They wanted to see if Kai was really a skilled furniture maker.D They wanted to make sure Kai's answers matched his father's. If a vulnerability is not fixed at the root cause, there is a possibility that another route of attack can emerge. This route is known as the ____________________. HELP ASAP?the two answers not showing on screen areC:y Help Meeeeeeeee. Have a nice day:) Companies that operate in a ___________________ environment have the lowest amount of environmental uncertainty as illustrated in the environmental uncertainty matrix. In the experiment on stimulation of the isolated sciatic nerve, a compound action potential is produced. This represents _______ A(n) _____ is the component of an expert system that performs tasks similar to what a human expert does by explaining to end users how recommendations are derived. A student walks 350 m [S], then 400 m [E20N], and finally 550 m [N10W]. Using the component method, find the resultant (total) displacement). Round your answer to the appropriate significant figures. Round your angle to the nearest degree. Find functions f(x) and g(x) so the given function can be expressed as h(x) = f(g(x)). (Use non-identity functions for f(x) and g(x).)h(x) = 5/x-4 A flywheel turns through 40 rev as it slows from an angular speed of 1.5 rad/s to a stop. (a) Assuming a constant angular acceleration, find the time for it to come to rest. (b) What is its angular acceleration The Purple Lion Beverage Company expects the following cash flows from its manufacturing plant in Palau over the next six years: Annual Cash Flows Year 1 Year 2 Year 3 Year 4 Year 5 Year 6 $250,000$37,500$180,000$300,000$750,000$725,000 The CFO of the company believes that an appropriate annual interest rate on this investment is 6.5%. What is the present value of this uneven cash flow stream, rounded to the nearest whole dollar What percent of the trees sampled were less than 75 ft tall? if salt and sand is mixed with distilled water, what will be the residue? and, what will be the filtrate?